LSAT and Law School Admissions Forum

Get expert LSAT preparation and law school admissions advice from PowerScore Test Preparation.

User avatar
 Dave Killoran
PowerScore Staff
  • PowerScore Staff
  • Posts: 5852
  • Joined: Mar 25, 2011
|
#43248
Complete Question Explanation
(The complete setup for this game can be found here: lsat/viewtopic.php?t=4568)

The correct answer choice is (A)

The condition in the question stem creates the following block:
PowerScore_LG_38_October_2002_game _#2_#11_diagram 1.png
Because this block places F after G, by applying the first rule we can infer that F’s other performance must come before the block:
PowerScore_LG_38_October_2002_game _#2_#11_diagram 2.png
Consequently, G’s other performance must come after F’s first performance, and thus either before or after the block. Adding the inference that L performs first creates the following scenario:
PowerScore_LG_38_October_2002_game _#2_#11_diagram 3.png
Thus, the only two possible performance orderings of volunteers are:
PowerScore_LG_38_October_2002_game _#2_#11_diagram 4.png
In both scenarios, F will demonstrate harvesting second, and thus answer choice (A) is correct.
You do not have the required permissions to view the files attached to this post.
 SherryZ
  • Posts: 124
  • Joined: Oct 06, 2013
|
#12071
Hi Dave,

Thank you for your help! However, I still don't understand your set up for Q11. You mentioned that "the only two possible performance orderings of volunteers are":


L-F-G-G-F-L
or
L-F-G-F-L-G

First, L-F-G-G-F-L seems violate the RULE 1 "F demonstrates exactly one task before G demonstrates any of the task" because of the "G-G". I interpret the RULE 1 as FG block.

In fact, I don't quite understand Q11's new rule "G demonstrates p immediately before F demonstrates t". Could you explain it to me?

Thank you!

--Sherry
User avatar
 Dave Killoran
PowerScore Staff
  • PowerScore Staff
  • Posts: 5852
  • Joined: Mar 25, 2011
|
#12073
Hi Sherry,

Let's turn this around again. You state that example 1 seems to violate that rule. But, I can tell you that it doesn't. So, where in your analysis do you think you went wrong?

The additional condition created in the question stem is separate from the issue above, and in this case a block is created:

  • ..... ..... P ..... T
    ..... ..... G ..... F

This block can then be linked to other formations in the game. Again, my ability to diagram this in the forum is limited. The explanation I referred you to above does a far more complete job :-D

Thanks!
 SherryZ
  • Posts: 124
  • Joined: Oct 06, 2013
|
#12080
Hi Dave,

At the beginning, I was wondering why can't L-F-G-F-G-L work? Then I reminded that "tm" must be a block (M must be after T). Based on:

G F
p t

L-F-G-F-G-L will become:

L-F-G-F-G-L
-h-p-t-m-

However, G CAN'T be h&m! So L-F-G-F-G-L cannot work. Does my analysis right??

In addition, I re-read the condition 1 again, it says "F demonstrates exactly one task before G demonstrates any of the tasks". Please correct me if I am wrong: does it mean F only needs to be ahead of G when G appears at FIRST TIME?

Thank you!!

---Sherry
User avatar
 Dave Killoran
PowerScore Staff
  • PowerScore Staff
  • Posts: 5852
  • Joined: Mar 25, 2011
|
#12086
Yes, basically. Once you have:


..... ..... P ..... T
..... ..... G ..... F


You also get because there is a TM block:


..... ..... P ..... T ..... M
..... ..... G ..... F


And M has to be L in this question. Thus, the solution here must feature G-F-L at some point.

Second, yes, that is the correct interpretation of the rule. One thing I've noticed from your posts is that you occasionally misinterpret rules. That is a point you should work on, because any time that happens you are likely to miss several questions at least. Words and phrases like "exactly" and "at least" make a huge difference in rules, so track them carefully.

Thanks and good luck!
 SherryZ
  • Posts: 124
  • Joined: Oct 06, 2013
|
#12118
Hi Dave,

Thank you very much for pointing my issue! I DO frequently misinterpret rules! Could you refer me to some pages or websites so that I can figure out the differences of words like "three chairs behind" and "behind three chairs"??

Thank you!!

Sincerely,
Sherry

Get the most out of your LSAT Prep Plus subscription.

Analyze and track your performance with our Testing and Analytics Package.